LSAT and Law School Admissions Forum

Get expert LSAT preparation and law school admissions advice from PowerScore Test Preparation.

User avatar
 Dave Killoran
PowerScore Staff
  • PowerScore Staff
  • Posts: 5850
  • Joined: Mar 25, 2011
|
#43437
Complete Question Explanation
(The complete setup for this game can be found here: lsat/viewtopic.php?t=16399)

The correct answer choice is (C)

According to the question stem, H and L are selected. From our discussion, we know that once H is selected, then K and N are also selected and F and T are not selected. This leaves the following situation:
J03_Game_#4_#19_diagram 1.png
Answer choices (A) and (B) are eliminated due to the Not Laws. Answer choices (D) and (E) are eliminated because all three pandas must be selected. Consequently, answer choice (C) is correct.

Get the most out of your LSAT Prep Plus subscription.

Analyze and track your performance with our Testing and Analytics Package.